Proof of an integral property [closed]

The name of the pictureThe name of the pictureThe name of the pictureClash Royale CLAN TAG#URR8PPP












4












$begingroup$


$$int_0^1 f(x)g'(x)dx=pi$$ If $f(1)g(1)=f(0)g(0)$ then $$int_0^1 f'(x)g(x)dx= -pi$$
So I have to prove this and I have absolutely no idea how to do it. I am guessing I will have to use the fundamental theorem of calculus and it show that the rate of change is $1$ because it didn't change from $f(1)g(1)$ to $f(0)g(0)$










share|cite|improve this question











$endgroup$



closed as off-topic by RRL, stressed out, Song, Cesareo, Kemono Chen Feb 22 at 0:58


This question appears to be off-topic. The users who voted to close gave this specific reason:


  • "This question is missing context or other details: Please provide additional context, which ideally explains why the question is relevant to you and our community. Some forms of context include: background and motivation, relevant definitions, source, possible strategies, your current progress, why the question is interesting or important, etc." – RRL, stressed out, Song, Cesareo, Kemono Chen
If this question can be reworded to fit the rules in the help center, please edit the question.











  • 8




    $begingroup$
    Do you know integration by parts?
    $endgroup$
    – Minus One-Twelfth
    Feb 21 at 1:37
















4












$begingroup$


$$int_0^1 f(x)g'(x)dx=pi$$ If $f(1)g(1)=f(0)g(0)$ then $$int_0^1 f'(x)g(x)dx= -pi$$
So I have to prove this and I have absolutely no idea how to do it. I am guessing I will have to use the fundamental theorem of calculus and it show that the rate of change is $1$ because it didn't change from $f(1)g(1)$ to $f(0)g(0)$










share|cite|improve this question











$endgroup$



closed as off-topic by RRL, stressed out, Song, Cesareo, Kemono Chen Feb 22 at 0:58


This question appears to be off-topic. The users who voted to close gave this specific reason:


  • "This question is missing context or other details: Please provide additional context, which ideally explains why the question is relevant to you and our community. Some forms of context include: background and motivation, relevant definitions, source, possible strategies, your current progress, why the question is interesting or important, etc." – RRL, stressed out, Song, Cesareo, Kemono Chen
If this question can be reworded to fit the rules in the help center, please edit the question.











  • 8




    $begingroup$
    Do you know integration by parts?
    $endgroup$
    – Minus One-Twelfth
    Feb 21 at 1:37














4












4








4





$begingroup$


$$int_0^1 f(x)g'(x)dx=pi$$ If $f(1)g(1)=f(0)g(0)$ then $$int_0^1 f'(x)g(x)dx= -pi$$
So I have to prove this and I have absolutely no idea how to do it. I am guessing I will have to use the fundamental theorem of calculus and it show that the rate of change is $1$ because it didn't change from $f(1)g(1)$ to $f(0)g(0)$










share|cite|improve this question











$endgroup$




$$int_0^1 f(x)g'(x)dx=pi$$ If $f(1)g(1)=f(0)g(0)$ then $$int_0^1 f'(x)g(x)dx= -pi$$
So I have to prove this and I have absolutely no idea how to do it. I am guessing I will have to use the fundamental theorem of calculus and it show that the rate of change is $1$ because it didn't change from $f(1)g(1)$ to $f(0)g(0)$







calculus integration definite-integrals






share|cite|improve this question















share|cite|improve this question













share|cite|improve this question




share|cite|improve this question








edited Feb 21 at 1:41









Eevee Trainer

7,96821439




7,96821439










asked Feb 21 at 1:36









adam hanyadam hany

212




212




closed as off-topic by RRL, stressed out, Song, Cesareo, Kemono Chen Feb 22 at 0:58


This question appears to be off-topic. The users who voted to close gave this specific reason:


  • "This question is missing context or other details: Please provide additional context, which ideally explains why the question is relevant to you and our community. Some forms of context include: background and motivation, relevant definitions, source, possible strategies, your current progress, why the question is interesting or important, etc." – RRL, stressed out, Song, Cesareo, Kemono Chen
If this question can be reworded to fit the rules in the help center, please edit the question.







closed as off-topic by RRL, stressed out, Song, Cesareo, Kemono Chen Feb 22 at 0:58


This question appears to be off-topic. The users who voted to close gave this specific reason:


  • "This question is missing context or other details: Please provide additional context, which ideally explains why the question is relevant to you and our community. Some forms of context include: background and motivation, relevant definitions, source, possible strategies, your current progress, why the question is interesting or important, etc." – RRL, stressed out, Song, Cesareo, Kemono Chen
If this question can be reworded to fit the rules in the help center, please edit the question.







  • 8




    $begingroup$
    Do you know integration by parts?
    $endgroup$
    – Minus One-Twelfth
    Feb 21 at 1:37













  • 8




    $begingroup$
    Do you know integration by parts?
    $endgroup$
    – Minus One-Twelfth
    Feb 21 at 1:37








8




8




$begingroup$
Do you know integration by parts?
$endgroup$
– Minus One-Twelfth
Feb 21 at 1:37





$begingroup$
Do you know integration by parts?
$endgroup$
– Minus One-Twelfth
Feb 21 at 1:37











2 Answers
2






active

oldest

votes


















5












$begingroup$

Hint:



Utilize integration by parts:



$$int f(x)g'(x)mathrm dx = f(x)g(x) - int f'(x)g(x) mathrm dx$$



If we have a definite integral, then this formula becomes



$$int_a^b f(x)g'(x)mathrm dx = f(b)g(b) - f(a)g(a) - int_a^b f'(x)g(x) mathrm dx$$






share|cite|improve this answer











$endgroup$








  • 1




    $begingroup$
    thank you so much for the answer ∫baf(x)g′(x)dx=−∫baf′(x)g(x)
    $endgroup$
    – adam hany
    Feb 21 at 2:01


















3












$begingroup$

Hint:



$$dfracd(f(x)cdot g(x))dx=?$$



Integrate both sides with respect to $x$ between $[0,1]$






share|cite|improve this answer











$endgroup$



















    2 Answers
    2






    active

    oldest

    votes








    2 Answers
    2






    active

    oldest

    votes









    active

    oldest

    votes






    active

    oldest

    votes









    5












    $begingroup$

    Hint:



    Utilize integration by parts:



    $$int f(x)g'(x)mathrm dx = f(x)g(x) - int f'(x)g(x) mathrm dx$$



    If we have a definite integral, then this formula becomes



    $$int_a^b f(x)g'(x)mathrm dx = f(b)g(b) - f(a)g(a) - int_a^b f'(x)g(x) mathrm dx$$






    share|cite|improve this answer











    $endgroup$








    • 1




      $begingroup$
      thank you so much for the answer ∫baf(x)g′(x)dx=−∫baf′(x)g(x)
      $endgroup$
      – adam hany
      Feb 21 at 2:01















    5












    $begingroup$

    Hint:



    Utilize integration by parts:



    $$int f(x)g'(x)mathrm dx = f(x)g(x) - int f'(x)g(x) mathrm dx$$



    If we have a definite integral, then this formula becomes



    $$int_a^b f(x)g'(x)mathrm dx = f(b)g(b) - f(a)g(a) - int_a^b f'(x)g(x) mathrm dx$$






    share|cite|improve this answer











    $endgroup$








    • 1




      $begingroup$
      thank you so much for the answer ∫baf(x)g′(x)dx=−∫baf′(x)g(x)
      $endgroup$
      – adam hany
      Feb 21 at 2:01













    5












    5








    5





    $begingroup$

    Hint:



    Utilize integration by parts:



    $$int f(x)g'(x)mathrm dx = f(x)g(x) - int f'(x)g(x) mathrm dx$$



    If we have a definite integral, then this formula becomes



    $$int_a^b f(x)g'(x)mathrm dx = f(b)g(b) - f(a)g(a) - int_a^b f'(x)g(x) mathrm dx$$






    share|cite|improve this answer











    $endgroup$



    Hint:



    Utilize integration by parts:



    $$int f(x)g'(x)mathrm dx = f(x)g(x) - int f'(x)g(x) mathrm dx$$



    If we have a definite integral, then this formula becomes



    $$int_a^b f(x)g'(x)mathrm dx = f(b)g(b) - f(a)g(a) - int_a^b f'(x)g(x) mathrm dx$$







    share|cite|improve this answer














    share|cite|improve this answer



    share|cite|improve this answer








    edited Feb 21 at 6:42









    mrtaurho

    6,00551641




    6,00551641










    answered Feb 21 at 1:39









    Eevee TrainerEevee Trainer

    7,96821439




    7,96821439







    • 1




      $begingroup$
      thank you so much for the answer ∫baf(x)g′(x)dx=−∫baf′(x)g(x)
      $endgroup$
      – adam hany
      Feb 21 at 2:01












    • 1




      $begingroup$
      thank you so much for the answer ∫baf(x)g′(x)dx=−∫baf′(x)g(x)
      $endgroup$
      – adam hany
      Feb 21 at 2:01







    1




    1




    $begingroup$
    thank you so much for the answer ∫baf(x)g′(x)dx=−∫baf′(x)g(x)
    $endgroup$
    – adam hany
    Feb 21 at 2:01




    $begingroup$
    thank you so much for the answer ∫baf(x)g′(x)dx=−∫baf′(x)g(x)
    $endgroup$
    – adam hany
    Feb 21 at 2:01











    3












    $begingroup$

    Hint:



    $$dfracd(f(x)cdot g(x))dx=?$$



    Integrate both sides with respect to $x$ between $[0,1]$






    share|cite|improve this answer











    $endgroup$

















      3












      $begingroup$

      Hint:



      $$dfracd(f(x)cdot g(x))dx=?$$



      Integrate both sides with respect to $x$ between $[0,1]$






      share|cite|improve this answer











      $endgroup$















        3












        3








        3





        $begingroup$

        Hint:



        $$dfracd(f(x)cdot g(x))dx=?$$



        Integrate both sides with respect to $x$ between $[0,1]$






        share|cite|improve this answer











        $endgroup$



        Hint:



        $$dfracd(f(x)cdot g(x))dx=?$$



        Integrate both sides with respect to $x$ between $[0,1]$







        share|cite|improve this answer














        share|cite|improve this answer



        share|cite|improve this answer








        edited Feb 21 at 6:09









        Eevee Trainer

        7,96821439




        7,96821439










        answered Feb 21 at 2:05









        lab bhattacharjeelab bhattacharjee

        227k15158275




        227k15158275












            Popular posts from this blog

            How to check contact read email or not when send email to Individual?

            Bahrain

            Postfix configuration issue with fips on centos 7; mailgun relay